La siguiente solución está tomada de Los módulos de un polinomio son iguales en AoPS. Para otras soluciones, véase Una familia de polinomios cuyos ceros se encuentran todos en el círculo unitario en Math Overflow.
La idea es derivar una fórmula de recursión para $$ f_n(z) = \sum_{k=0}^{n} \binom{n}{k}\lambda^{k(n-k)} z^k $$ y demostrar la afirmación por inducción. $\lambda$ es un número real fijo en el intervalo $(0, 1)$ .
$f_0(z) = 1$ y $f_1(z) = 1+z$ seguramente sólo tienen raíces de módulo uno.
Supongamos ahora que $n \ge 1$ y todas las raíces de $f_n$ tienen módulo uno. Utilizando la fórmula de recurrencia para los coeficientes binomiales $$ \binom{n+1}{k} = \binom{n}{k} + \binom{n}{k-1} $$ se obtiene $$ f_{n+1}(z) = f_n(\lambda z) + \lambda^n z f_n(\frac{z}{\lambda}) \, . $$ Ahora dejemos que $z^*$ sea un cero de $f_{n+1}$ . Tenemos que demostrar que $|z^*| = 1$ . De la fórmula de recurrencia anterior obtenemos que $$ f_n(\lambda z^*) = - \lambda^n z^* f_n(\frac{z^*}{\lambda}) $$ Tenga en cuenta que $f_n(\frac{z^*}{\lambda})$ no puede ser cero: En caso contrario, ambos $\lambda z^*$ y $\frac{z^*}{\lambda}$ tendría módulo uno, lo que no es posible.
Denotando las raíces de $f_n$ con $z_1, \ldots z_n$ tenemos $f_n(z) = (z-z_1) \cdots (z-z_n)$ para que $$ z^* = - \frac{f_n(\lambda z^*)}{\lambda^n f_n(\frac{z^*}{\lambda})} = - \prod_{k=1}^n \frac{\lambda z^* - z_k}{z^* - \lambda z_k} $$ y por lo tanto $$ \tag{*} |z^*| = \prod_{k=1}^n \left |\frac{\lambda z^* - z_k}{z^* - \lambda z_k} \right | \, . $$ Un cálculo elemental (utilizando $|z_k|=1$ para todos $k$ ) muestra que $$ |\lambda z^* - z_k|^2 - |z^* - \lambda z_k|^2 = (1 - \lambda^2)(1 - |z^*|^2) \, . $$
De ello se deduce que si $|z^*| < 1$ entonces todos los factores del lado derecho de $(*)$ son mayores que uno, y si $|z^*| > 1$ entonces todos los factores del lado derecho de $(*)$ son menores que uno. Así que ambos casos conducen a una contradicción, y la única posibilidad es que $|z^*| = 1$ . Con esto concluye la prueba.
4 votos
El polinomio tiene coeficientes "simétricos" y por lo tanto se puede escribir como $g(\frac{x+\frac{1}{x}}{2})$ donde $g$ es un polinomio. Entonces el problema se reduce a demostrar que $g$ tiene todas sus raíces reales y en el interior $[-1,1]$ . En este caso, las técnicas, como en la pregunta de Putnam mencionada en el PO, probablemente serían útiles.
2 votos
También publicado en MO: mathoverflow.net/questions/299304/
1 votos
@EwanDelanoy Bueno, $x+1$ tiene coeficientes "simétricos" pero cómo escribirlo como un polinomio de $\dfrac12\left(x+\dfrac1x\right)$ ?
0 votos
@AlexFrancisco Se trata de un ejercicio clásico : por inducción sobre $n$ se demuestra que $x^n+\frac{1}{x^n}$ es de la forma $Q_n(x+\frac{1}{x})$ donde $Q_n$ es un polinomio de grado único definido $n$ . El término exacto es "palindrómico", por cierto.
0 votos
@EwanDelanoy ¿No es esto cierto sólo para polinomios de grado par?
0 votos
@AlexFrancisco Acabo de darme cuenta de que olvidé un factor de escala en mi fórmula : Debería haber dicho $f(x)=(x^{\deg(f)})g(\frac{x+\frac{1}{x}}{2})$ .
0 votos
@Servaes Este trabajo en el grado impar también : por ejemplo $x^1+\frac{1}{x^1}=Q_1(x+\frac{1}{x})$ donde $Q_1(t)=t$ y $x^3+\frac{1}{x^3}=Q_3(x+\frac{1}{x})$ donde $Q_3(t)=t^3-3t$ etc.
0 votos
@EwanDelanoy Tampoco $x^1+\frac{1}{x^1}$ ni $x^3+\frac{1}{x^3}$ son polinomios, sin embargo, y su factor de escala producirá polinomios de grado par.
0 votos
@Servaes no entiendo para nada tu último comentario. En ningún momento he dicho $x^n+\frac{1}{x^n}$ era un polinomio, y le dio ejemplos de grado impar : $Q_1$ y $Q_3$ .
0 votos
@EwanDelanoy Volviendo al punto original, ¿qué quiere decir con "simétrico"? ¿Cómo escribirías $f(x)=x+1$ como $f(x)=x^{\deg(f)}g(\frac{x+\frac{1}{x}}2)$ ?
0 votos
@Servaes $f$ es palindrómico cuando $f(x)=\sum_{k=0}^n a_kx^k$ con $a_k=a_{n-k}$ por cada $k$ . Sin embargo, tienes razón, mi descomposición sólo funciona para las $n$ (aunque $g$ puede ser de cualquier grado). Y me he vuelto a equivocar con la fórmula: debería ser $f(x)=x^{\frac{\deg(f)}{2}}g(\frac{x+\frac{1}{x}}{2})$ $f(x)=x^{\deg(f)}g(\frac{x+\frac{1}{x}}{2})$
0 votos
Creo que esto se responde en MO. Es necesario poner un resumen aquí?